LSAT and Law School Admissions Forum

Get expert LSAT preparation and law school admissions advice from PowerScore Test Preparation.

 JessicaM
  • Posts: 3
  • Joined: Dec 11, 2020
|
#82987
JessicaM wrote: Tue Jan 05, 2021 4:57 am I can't see why D does not weaken
^^ To add to my question, is the conclusion that "there will be an increase in factory jobs" or that "the minimum wage increase will lead to decrease unemployment?" But yes, also still confused about D.
User avatar
 KelseyWoods
PowerScore Staff
  • PowerScore Staff
  • Posts: 1079
  • Joined: Jun 26, 2013
|
#83021
Hi Jessica!

The conclusion of this argument is the first sentence: "The increase in the minimum wage in Country X will quickly lead to a decrease in Country X’s rate of unemployment." The subsequent sentences explain why the increase in the minimum wage will lead to a decrease in the rate of unemployment.

Notice that this is a Weaken EXCEPT question. That means that the four incorrect answers will weaken the argument and we are looking for the one correct answer which does NOT weaken. So answer choice (D) does weaken the argument, which is why it is incorrect--we're looking for an answer choice that does NOT weaken it. Answer choices (A), (B), (D), and (E) all weaken the argument. Answer choice (C) is the one which does not weaken the argument.

As for why answer choice (C) does not weaken the argument, check out Paul's post above:
(C) only tells us that some factory workers are paid more than minimum wage. Like Adam mentioned above, this ignores both other factory workers (who do make minimum wage) as well as all non-factory workers. But just as importantly, the premises of the stimulus already tell us that "Raising the minimum wage will lead to more disposable income for a large segment of the working population". (As always, for Weaken questions we're not worried about attacking the premises, rather we're finding an issue with how the conclusion is drawn out of those premises). So since we've already established that raising the minimum wage will mean more $ for a large segment of workers, who cares if some factory workers aren't affected? That doesn't weaken the argument.
Hope this helps!

Best,
Kelsey
User avatar
 JocelynL
  • Posts: 51
  • Joined: Dec 22, 2020
|
#83636
Hello,
I thought the last sentence was the author's conclusion. "Surely this increase in demand for consumer goods will lead to an increase in the number of factory jobs necessary to meet production"
C - increase in demand for goods
E - in crease in the number of factory jobs necessary to meet production

A removes the cause by stating that there will be higher prices for consumer goods which will not result in an increase in demand for goods.

B, D, and E all remove the authors stated effect of an "increase in the number of jobs necessary", which weaken the statement.

However, I'm concerned that I didn't correctly identify the argument in this stimulus. I'm struggling with this overall. Any feedback in my reasonings above for eliminating the a/c or help in correctly identifying the argument will be very helpful.
 Robert Carroll
PowerScore Staff
  • PowerScore Staff
  • Posts: 1787
  • Joined: Dec 06, 2013
|
#83640
Jocelyn,

The last sentence is just showing specifically how the increase in the minimum wage will impact a particular kind of employment. That's intended to serve the main point in the first sentence, which is that unemployment overall will decrease. To see that the first sentence is the conclusion, consider - if it's not, what role does it have in the argument? The author isn't trying to use it as a premise - it's not intended to show that the last sentence is true, for instance. It's not possible to locate the first sentence's role in the argument unless it's the conclusion. Thus, the last sentence can be seen to be the completion of a chain of reasoning that intends to show that factory jobs will increase. That being shown, the argument has now made its case that employment overall will increase, because factory jobs will help that overall increase.

Your evaluations of the answer choices are perfect, though. Because the argument is relying on an increase in factory jobs to produce an increase in employment overall, anything that cuts away the support for the increase in factory jobs will also weaken the argument.

Robert Carroll
User avatar
 shanhickey
  • Posts: 21
  • Joined: Apr 11, 2022
|
#95372
Hello!

Can someone please explain why A weakens? I was thinking that even if minimum wage increased and prices increased it doesn't necessarily hurt the argument because it doesn't take away the fact that employment will increase and people will have more disposable income. In fact, if prices are going up, wouldn't people end up spending more since the price is increased? I chose A over C. C didn't really register for me when I was taking this timed. I'm just confused.

Thanks!!
 Adam Tyson
PowerScore Staff
  • PowerScore Staff
  • Posts: 5153
  • Joined: Apr 14, 2011
|
#95432
If my wages go up by, say, 15%, shanhickey, and prices also go up by 15%, then I will be no wealthier than I was before in terms of my buying power. I won't be able to buy more things than I did before, so there won't be any increase in demand for consumer goods, and thus no increase in factory jobs to produce more goods!

We might get the predicted result if wages go up more than prices rise, but we also might get the opposite result if prices rise faster than wages do, so this answer at least raises the possibility of not getting the predicted result, and that element of doubt is enough to weaken the argument.
 lydia.grace1468@gmail.com
  • Posts: 2
  • Joined: Oct 15, 2022
|
#103339
Hi I selected A because I thought it was neutral to the argument and thereby weakened it. Could you explain why A is relevant and or weakens the argument?

An explanation for all the answer choices would be nice as well!
 lydia.grace1468@gmail.com
  • Posts: 2
  • Joined: Oct 15, 2022
|
#103341
lydia.grace1468@gmail.com wrote: Tue Sep 26, 2023 12:40 pm Hi I selected A because I thought it was neutral to the argument and thereby weakened it. Could you explain why A is relevant and or weakens the argument?

An explanation for all the answer choices would be nice as well!
Nevermind! I didn't know there were multiple pages I found the answer.
User avatar
 hjoon514
  • Posts: 1
  • Joined: Nov 19, 2023
|
#104031
I dont get why (A) would weaken the passage TT can someone help

Get the most out of your LSAT Prep Plus subscription.

Analyze and track your performance with our Testing and Analytics Package.